A Module $M$ is Irreducible if and only if $M$ is isomorphic to $R/I$ for a Maximal Ideal $I$.

Module Theory problems and solutions

Problem 449

Let $R$ be a commutative ring with $1$ and let $M$ be an $R$-module.
Prove that the $R$-module $M$ is irreducible if and only if $M$ is isomorphic to $R/I$, where $I$ is a maximal ideal of $R$, as an $R$-module.

 
LoadingAdd to solve later

Sponsored Links


 

Definition (Irreducible module).

An $R$-module $M$ is called irreducible if $M$ is not the zero module and $0$ and $M$ are the only submodules of $M$.

An irreducible module
is also called a simple module.

Proof.

$(\implies)$ Suppose that $M$ is an irreducible $R$-module.
Then by definition of an irreducible module, $M$ is not the zero module.
Take any nonzero element $m\in M$, and consider the cyclic submodule $(m)=Rm$ generated by $m$.
Since $M$ is irreducible, we must have $M=Rm$.

Now we define a map $f:R\to M$ by sending $r\in R$ to $f(r)=rm$.
Then the map $f$ is an $R$-module homomorphism regarding $R$ is an $R$-module.

In fact, we have
\begin{align*}
&f(r+s)=(r+s)m=rm+sm=f(r)+f(s)\\
&f(rs)=(rs)m=r(sm)=rf(s)
\end{align*}
for any $r, s\in R$.
Since $M=Rm$, the homomorphism $f$ is surjective.
Thus, by the first isomorphism theorem, we obtain
\[R/I\cong M,\] where $I=\ker(f)$.

It remains to show that $I$ is a maximal ideal of $R$.
Suppose that $J$ is an ideal such that $I\subset J \subset R$.
Then by the third isomorphism theorem for rings, we know that $J/I$ is an ideal of the ring $R/I\cong M$, hence $J/I$ is a submodule.

Since $M$ is irreducible, we must have either $J/I=0$ or $J/I=M$.
This implies that $J=I$ or $J=M$.
Hence $I$ is a maximal ideal.


$(\impliedby)$ Suppose now that $M\cong R/I$ for some maximal ideal $I$ of $R$.
Let $N$ be any submodule of $R/I$. (We identified $M$ and $R/I$ by the above isomorphism.)
Then $N$ is an ideal of $R/I$ since $N$ is an abelian group and closed under the action of $R$, hence that of $R/I$.

Since $R$ is a commutative ring and $I$ is a maximal ideal of $R$, we know that $R/I$ is a field.
Thus, only the ideals of $R/I$ are $0$ or $R/I$.

Hence we have $N=0$ or $N=R/I=M$.
This proves that $M$ is irreducible.

Related Question.

A similar technique in the proof above can be used to solve the following problem.

Problem. Let $R$ be a ring with $1$.
Prove that a nonzero $R$-module $M$ is irreducible if and only if $M$ is a cyclic module with any nonzero element as its generator.

See the post “A module is irreducible if and only if it is a cyclic module with any nonzero element as generator” for a proof.


LoadingAdd to solve later

Sponsored Links

More from my site

  • A Module is Irreducible if and only if It is a Cyclic Module With Any Nonzero Element as GeneratorA Module is Irreducible if and only if It is a Cyclic Module With Any Nonzero Element as Generator Let $R$ be a ring with $1$. A nonzero $R$-module $M$ is called irreducible if $0$ and $M$ are the only submodules of $M$. (It is also called a simple module.) (a) Prove that a nonzero $R$-module $M$ is irreducible if and only if $M$ is a cyclic module with any nonzero element […]
  • Torsion Submodule, Integral Domain, and Zero DivisorsTorsion Submodule, Integral Domain, and Zero Divisors Let $R$ be a ring with $1$. An element of the $R$-module $M$ is called a torsion element if $rm=0$ for some nonzero element $r\in R$. The set of torsion elements is denoted \[\Tor(M)=\{m \in M \mid rm=0 \text{ for some nonzero} r\in R\}.\] (a) Prove that if $R$ is an […]
  • Annihilator of a Submodule is a 2-Sided Ideal of a RingAnnihilator of a Submodule is a 2-Sided Ideal of a Ring Let $R$ be a ring with $1$ and let $M$ be a left $R$-module. Let $S$ be a subset of $M$. The annihilator of $S$ in $R$ is the subset of the ring $R$ defined to be \[\Ann_R(S)=\{ r\in R\mid rx=0 \text{ for all } x\in S\}.\] (If $rx=0, r\in R, x\in S$, then we say $r$ annihilates […]
  • Submodule Consists of Elements Annihilated by Some Power of an IdealSubmodule Consists of Elements Annihilated by Some Power of an Ideal Let $R$ be a ring with $1$ and let $M$ be an $R$-module. Let $I$ be an ideal of $R$. Let $M'$ be the subset of elements $a$ of $M$ that are annihilated by some power $I^k$ of the ideal $I$, where the power $k$ may depend on $a$. Prove that $M'$ is a submodule of […]
  • Ascending Chain of Submodules and Union of its SubmodulesAscending Chain of Submodules and Union of its Submodules Let $R$ be a ring with $1$. Let $M$ be an $R$-module. Consider an ascending chain \[N_1 \subset N_2 \subset \cdots\] of submodules of $M$. Prove that the union \[\cup_{i=1}^{\infty} N_i\] is a submodule of $M$.   Proof. To simplify the notation, let us […]
  • Finitely Generated Torsion Module Over an Integral Domain Has a Nonzero AnnihilatorFinitely Generated Torsion Module Over an Integral Domain Has a Nonzero Annihilator (a) Let $R$ be an integral domain and let $M$ be a finitely generated torsion $R$-module. Prove that the module $M$ has a nonzero annihilator. In other words, show that there is a nonzero element $r\in R$ such that $rm=0$ for all $m\in M$. Here $r$ does not depend on […]
  • Linearly Dependent Module Elements / Module Homomorphism and Linearly IndependencyLinearly Dependent Module Elements / Module Homomorphism and Linearly Independency (a) Let $R$ be a commutative ring. If we regard $R$ as a left $R$-module, then prove that any two distinct elements of the module $R$ are linearly dependent. (b) Let $f: M\to M'$ be a left $R$-module homomorphism. Let $\{x_1, \dots, x_n\}$ be a subset in $M$. Prove that if the set […]
  • Nilpotent Ideal and Surjective Module HomomorphismsNilpotent Ideal and Surjective Module Homomorphisms Let $R$ be a commutative ring and let $I$ be a nilpotent ideal of $R$. Let $M$ and $N$ be $R$-modules and let $\phi:M\to N$ be an $R$-module homomorphism. Prove that if the induced homomorphism $\bar{\phi}: M/IM \to N/IN$ is surjective, then $\phi$ is surjective.   […]

You may also like...

1 Response

  1. 06/10/2017

    […] For a proof of this problem, see the post “A Module $M$ is Irreducible if and only if $M$ is isomorphic to $R/I$ for a Maximal Ideal $I$.“. […]

Leave a Reply

Your email address will not be published. Required fields are marked *

This site uses Akismet to reduce spam. Learn how your comment data is processed.

More in Module Theory
Module Theory problems and solutions
A Module is Irreducible if and only if It is a Cyclic Module With Any Nonzero Element as Generator

Let $R$ be a ring with $1$. A nonzero $R$-module $M$ is called irreducible if $0$ and $M$ are the...

Close